Your LG Approach - June 94 Q18 Forum

Prepare for the LSAT or discuss it with others in this forum.
Post Reply
User avatar
LSAT Taker

Bronze
Posts: 185
Joined: Mon Oct 26, 2009 5:01 pm

Your LG Approach - June 94 Q18

Post by LSAT Taker » Sun Apr 25, 2010 4:12 pm

I am not sure about the PT number for this, but this is the “housing committee with a chairperson” question. I wonder what your approach to this question was.

I now understand why A is the answer, but only after applying every answer choice to the set up. Frankly, it also took me several minutes to understand what the question is really asking. For those of you who got it right away, how did you see the point? After all, it is not very common that an LG question is followed by 5 answer choices that are all conditional statements.

Thanks, and good luck to all June LSAT takers!

User avatar
Atlas LSAT Teacher

Bronze
Posts: 283
Joined: Tue May 12, 2009 10:18 am

Re: Your LG Approach - June 94 Q18

Post by Atlas LSAT Teacher » Sun Apr 25, 2010 6:10 pm


User avatar
LSAT Taker

Bronze
Posts: 185
Joined: Mon Oct 26, 2009 5:01 pm

Re: Your LG Approach - June 94 Q18

Post by LSAT Taker » Mon Apr 26, 2010 8:14 am

Thanks for this! Actually, the set up you said your friend used looks almost identical to mine. I, however, still struggled with the question with 5 conditional statements (Other questions were relatively easy.). Could that be simply because I freaked out by rather an unusual angle of the question?

User avatar
matt@atlaslsat

New
Posts: 63
Joined: Wed Mar 31, 2010 2:34 pm

Re: Your LG Approach - June 94 Q18

Post by matt@atlaslsat » Mon Apr 26, 2010 12:36 pm

So, I thought I would just add my two cents on this.

The rule regarding J and M is too strong to not utilize. Since J is selected if, and only if, M is selected then either J and M are both selected or neither J and M are selected, creating two hypotheticals when the chairperson is a homeowner.

I don't see where to attach a slide, so I've posted one at the following link.

http://www.atlaslsat.com/forums/pt11-s1 ... -t763.html

I didn't explain each step, but I think if you start with the constraint regarding J and M and then follow the rules, you'll see each inference in the setup.

Let me know if this doesn't make this question much easier. Also, when I approach this game, I have 4 frames almost entirely filled out. I don't have to do any more writing once I have them established. I base them off two splits. The first split is that there are either 2 tenants and 3 homeowners, or 3 tenants and 2 homeowners. Then for each, either J and M are both selected or neither are selected. This creates 4 frames with every possible solution before I even start with the questions.

As Yogi Berra once said, "when you come to a fork in the road, take it!"

User avatar
LSAT Taker

Bronze
Posts: 185
Joined: Mon Oct 26, 2009 5:01 pm

Re: Your LG Approach - June 94 Q18

Post by LSAT Taker » Tue Apr 27, 2010 8:46 am

Many thanks for this!!!

Want to continue reading?

Register now to search topics and post comments!

Absolutely FREE!


Post Reply

Return to “LSAT Prep and Discussion Forum”